Difference between revisions of "2001 IMO Shortlist Problems/A6"

m (rearranged format)
Line 9: Line 9:
  
 
== Solution ==
 
== Solution ==
{{solution}}
+
We will use the Jenson's inequality.
  
 +
Now, normalize the inequality by assuming <math>a+b+c=1</math>
 +
 +
Consider the function <math>f(x)=\frac{1}{\sqrt{x}}</math>. Note that this function is convex and monotonically decreasing which implies that if <math>a > b</math>, then <math>f(a) < f(b)</math>.
 +
 +
Thus, we have
 +
 +
<math>\frac {a}{\sqrt {a^2 + 8bc}} + \frac {b}{\sqrt {b^2 + 8ca}} + \frac {c}{\sqrt {c^2 + 8ab}} = af(a^2+8bc)+bf(b^2+8ca)+cf(c^2+8ab) \geq f(a^3+b^3+c^3+24abc)</math>
 +
 +
Thus, we only need to show that <math>a^3+b^3+c^3+24abc \leq 1</math> i.e.
 +
 +
<cmath>a^3+b^3+c^3+24abc \leq (a+b+c)^3=a^3+b^3+c^3+3(ab+bc+ca)-3abc</cmath>
 +
 +
<cmath>i.e.  (ab+bc+ca) \geq 9abc</cmath>
 +
 +
Which is true since
 +
 +
<cmath>(ab+bc+ca)=(a+b+c)(ab+bc+ca)=(a+b)(b+c)(c+a)+abc \geq 2\sqrt{ab}.2\sqrt{bc}.2\sqrt{ca}+abc=9abc</cmath>
 +
The last part follows by the AM-GM inequality.
 +
 +
Equality holds if <math>a=b=c</math>
  
 
== Resources ==
 
== Resources ==

Revision as of 12:01, 1 September 2017

Problem

Prove that for all positive real numbers $a,b,c$,

$\frac {a}{\sqrt {a^2 + 8bc}} + \frac {b}{\sqrt {b^2 + 8ca}} + \frac {c}{\sqrt {c^2 + 8ab}} \geq 1.$

Generalization

The leader of the Bulgarian team had come up with the following generalization to the inequality:

$\frac {a}{\sqrt {a^2 + kbc}} + \frac {b}{\sqrt {b^2 + kca}} + \frac {c}{\sqrt {c^2 + kab}} \geq \frac{3}{\sqrt{1+k}}.$

Solution

We will use the Jenson's inequality.

Now, normalize the inequality by assuming $a+b+c=1$

Consider the function $f(x)=\frac{1}{\sqrt{x}}$. Note that this function is convex and monotonically decreasing which implies that if $a > b$, then $f(a) < f(b)$.

Thus, we have

$\frac {a}{\sqrt {a^2 + 8bc}} + \frac {b}{\sqrt {b^2 + 8ca}} + \frac {c}{\sqrt {c^2 + 8ab}} = af(a^2+8bc)+bf(b^2+8ca)+cf(c^2+8ab) \geq f(a^3+b^3+c^3+24abc)$

Thus, we only need to show that $a^3+b^3+c^3+24abc \leq 1$ i.e.

\[a^3+b^3+c^3+24abc \leq (a+b+c)^3=a^3+b^3+c^3+3(ab+bc+ca)-3abc\]

\[i.e.   (ab+bc+ca) \geq 9abc\]

Which is true since

\[(ab+bc+ca)=(a+b+c)(ab+bc+ca)=(a+b)(b+c)(c+a)+abc \geq 2\sqrt{ab}.2\sqrt{bc}.2\sqrt{ca}+abc=9abc\] The last part follows by the AM-GM inequality.

Equality holds if $a=b=c$

Resources